There are 4 colas, 1 ginger, 7 root beers, and 6 cherry sodas in a cooler. What are the odds of choosing a ginger ale? Give your answer in a proportion in lower terms.

Answers

Answer 1

Answer:

Step-by-step explanation:

the odds of choosing a ginger ale is 1/21

Answer 2
The odds of choosing a ginger ale is 1/18.

To find the proportion, add all the numbers together, and put that as the denominator. Put the number that you are trying to find the proportion of as the numerator

Related Questions

graph a
graph b
graph c
graph d

Answers

which of the following isthe√×51? answer is A .0-1

Find the height of the triangle by applying formulas for the area of a triangle and your knowledge about triangles.
A. 10.5 cm
B. 3.4 cm
C. 8.5 cm
D. 12 cm

Answers

Answer:

12 cm is the right answer pls mark me brainliest

The height of the triangle by applying formulas for the area of a triangle and your knowledge about triangles is 12 cm.

What is Area of Triangle?

The area of a triangle is defined as the total space occupied by the three sides of a triangle in a 2-dimensional plane. The basic formula for the area of a triangle is equal to half the product of its base and height, i.e., A = 1/2 × b × h.

What is Heron's formula?

Heron's formula, formula credited to Heron of Alexandria (c. 62 ce) for finding the area of a triangle in terms of the lengths of its sides. In symbols, if a, b, and c are the lengths of the sides:

Area = √s(s - a)(s - b)(s - c) where s is half the perimeter, or (a + b + c)/2.

Given:

Three sides are: 15cm, 25 cm and 2 cm

Now, Using Heron's formula

semi-perimeter= (25+ 20 + 15)/2

s= 30 cm

Now,

Area of triangle

=√s(s-a)(s-b)(s-c)

=√30* 5 * 10* 15

=√5*2*3*5*2*5*3*5

=5*5*2*3

=150 cm²

Again, area of triangle= 1/2* b* h

150= 1/2* 25* x

12cm= x

Learn more about Area of Triangle here:

https://brainly.com/question/9817285

#SPJ2

what is the value of a in the functions equation?

Answers

Answer:

y= -2x^2 + 12x -14

Step-by-step explanation:

when I put the points into a graphing calculator this is what I got so I think the answer is -2

Answer:

Step-by-step explanation:

It is -2

Cory remembers that his ATV had 4 gallons of gasoline in the tank on Monday. After driving a total of 40 miles during
the week, he has 2 gallons of gas remaining. Find the slope of the graph representing this situation.

Answers

Answer:

-1/20

Step-by-step explanation:

For a graph like this you should make the gallons of gas on y-axis and the miles driven on the x-axis.

To find slope the formula is (y2-y1)/(x2-x1)

So in this case it is 2-4/40-0

-2/40

This reduces to -1/20

Which of the following is true? Tangent is positive in Quadrant I. Sine is negative in Quadrant II. Cosine is positive in Quadrant III. Sine is positive in Quadrant IV.

Answers

A) Tangent is positive in Quadrant I.

Since sine and cosine are both positive in Quadrant I and tangent is the ratio of sine to cosine, tangent is positive in Quadrant I

Answer:

A

Step-by-step explanation:

I had this question and got it right the user above explains it in detail

1. 3x + 6y = 3 and 7x + 3y = 7
ons for bo​

Answers

Answer:

(1,0)

Step-by-step explanation:

3x + 6y = 3

7x + 3y = 7

Multiply the second equation by -2

-2( 7x + 3y = 7)

-14x -6y = -14

Add this to the first equation to eliminate y

3x + 6y = 3

-14x -6y = -14

--------------------

-11 x = -11

Divide by -11

x = 1

Now find y

3x + 6y = 3

3 +6y = 3

Subtract 3 from each side

6y = 0

y =0

Answer:

x = 1

y = 0

Step-by-step explanation:

3x + 6y = 3

7x  + 3y = 7

=> 3y = 7 - 7x

=> y = -7/3x + 7/3

3x + 6(-7/3x + 7/3) = 3

=> 3x - 14x + 14 = 3

=> -11x = -11

=> -x = -11/11

=> -x = -1

=> x = 1

So, 3(1) + 6y = 3

=> 3 + 6y = 3

=> 6y = 0

=> y = 0/6

=> y = 0

So, x = 1

      y = 0

Which of the following is an irrational number?
5 / 4
√5 / 7
1/ 8
3 / 5

Answers

Answer:  [tex]\sqrt{5} /7[/tex]

Step-by-step explanation:

5/4 is not an irrational number because it is already in a fraction the same as 1/8 and 3/5.

The square root of 5 is not rational because it cannot be converted to a fraction or in other words is not a perfect square.

Answer:√5 / 7

Step-by-step explanation:

In which direction must the graph of Ax) = x be shifted to produce the graph of g(x) = f(x) - 4?
ОА. up
OB. down
O c. left and down
OD. right and up​

Answers

Answer: B. down

Step-by-step explanation:

Translation rules:

For a function h(x):

h(x+c) is a left-shift by c units.h(x-c) is a right-shift by c units.h(x)+c is a up-shift by c units.h(x)-c is a down-shift by c units.

Here, the graph of f(x) becomes the graph of g(x) =f(x)-4 which is similar to "h(x)-c".

That means , f(x) is shifted 4 units down to become g(x).

So, correct option : B. down

If the measure of angle 4 is (11 x) degrees and angle 3 is (4 x) degrees, what is the measure of angle 3 in degrees?

Answers

Answer:

is it 2

Step-by-step explanation:

on a map, the distance between jacksonville FL and tallahasse FL is about 5 inches. According to the scale, 1 inch represents 25 miles. About how far apart are these two cities?

Answers

Jacksonville Fl and Tallahassee FL will be about 125 miles apart in real life.

complete the square to solve
f(x)=x^2-6x+5​

Answers

Answer: See below

Explanation:

x^2 - 6x + 5 = 0
(x^2 - 6x + 9) + 5 - 9 = 0
(x - 3)(x - 3) - 4 = 0
(x - 3)^2 = 4
x - 3 = +-2 (sqrt both sides)

x - 3 = 2
x = 5

x - 3 = -2
x = 1

Help the question is there

Answers

Answer:

y = 7 when x = -5

Step-by-step explanation:

First go to x = -5

Then go up to where you meet the green line

The y value is 7

y = 7 when x = -5

Georgina has $5.60 in quarters and dimes. If Georgina has 14 more dimes than quarters, how many quarters does she have?
[I don't want the answer i just want someone to show me how the problem is set up because its confusing me...Thanks]

Answers

Answer:

She has 12 quarters.

Step-by-step explanation:

You have two unknowns, the number of quarters and the number of dimes. You need one equation per unknown, so you need two equations. One equation deals with the number of coins. The other equation deals with the values of the coins.

The first step is to choose variables. We can go with x and y, but I prefer to choose q and d, so I remember more easily what they represent.

Let q = number of quarters.

Let d = number of dimes.

Let's deal with the numbers of coins first.

"Georgina has 14 more dimes than quarters"

She has q number of quarters, but the number of dimes is 14 more than the number of quarters. d is 14 more than q. Our first equation is

d = q + 14

Now we deal with the values of the coins.

We don't know yet the actual numbers of dimes and quarters, so we use d and q to represent those numbers.

One dime is worth $0.10; d number of dimes is worth 0.1d.

A quarter is worth $0.25; q number of quarters is worth 0.25q.

The total value of the dimes and quarters is the sum of the values of the two sets of coins:

0.1d + 0.25q

We are told "Georgina has $5.60 in quarters and dimes."

This gives us our second equation.

0.1d + 0.25q = 5.6

Now we have a set of two equations in two variables:

d = q + 14

0.1d + 0.25q = 5.6

There are several methods for solving a system of equations. Since the first equation is already solved for a variable, d, we can use the substitution method.

We rewrite the second equation, but we substitute q + 14 for d in the second equation.

Second equation:

0.1d + 0.25q = 5.6

Substitute q + 14 for d:

0.1(q + 14) + 0.25q = 5.6

Distribute on the left side:

0.1q + 1.4 + 0.25q = 5.6

Combine like terms on the left side:

0.35q + 1.4 = 5.6

Subtract 1.4 from both sides:

0.35q = 4.2

Divide both sides by 0.35:

q = 12

There are 12 quarters.

(You are asked only the number of quarters, so you can stop here. I will continue to find also the number of dimes for 2 reasons. 1) You see how it's done, so it will help with other problems. 2) By finding the numbers of dimes and quarters, then we can check if our solution is correct, which I will do below at the end.)

Now we use the first equation, d = q + 14. We substitute 12 for q and solve for d.

d = q + 14

d = 12 + 14

d = 26

There are 26 dimes.

Check:

We check the numbers of coins:

26 - 12 = 14 The number of dimes is indeed 14 more than the number of quarters.

We check the values of the coins:

0.1(26) + 0.25(12) = 2.6 + 3 = 5.6 The value of the coins is indeed $5.60.

Our answer is correct.

The projected worth (in millions of dollars) of a large company is modeled by the equation w = 236(1.06) t. The variable t represents the number of years since 2000. What is the projected annual percent of growth, and what should the company be worth in 2011? A. 6%; $448.00 million B. 16%; $474.88 million C. 16%; $250.16 million D. 6%; $422.64 million

Answers

Answer:

  A.  6%, $448 million

Step-by-step explanation:

a) The base of the exponential term is 1.06, so the projected annual growth is 1.06 -1 = .06 = 6%

__

b) Filling in 11 for t, we find the projected worth to be ...

  w = 236(1.06^11) ≈ $448 . . . million

Please answer quickly! A radio telescope has a parabolic surface, as shown below. A parabola opening up with vertex at the origin is graphed on the coordinate plane. The height of the parabola from top to bottom is 1 meter and its width from left to right is 20 meters. If the telescope is 1 m deep and 20 m wide, how far is the focus from the vertex?

Answers

Answer:

Basing on the description, a parabola checking with vertex at origin, the formula with vertex at origin can be used, x^2 = 4py. p is the focus therefore with the dimensions given, we get yourself a 0.25 and this is the distance of the focus to the vertex.

I need help ASAP!!

Can someone explain this? And answer it? I am so confused!!

Answers

Answer:

Step-by-step explanation: hope this helps

Will Give Brainliest, answer ASAP x=
y=
z=

Answers

Opposite angles are identical.

11y = 55

Y = 55/11

Y = 5

3z-4 = 110

Add 4 to both sides

3z= 114

Z = 114/3

Z = 38

180 -110 = 70

55 + x = 70

X = 70-55

X =15

X = 15, y = 5, z = 38

Answer:x=15, y=5, z=38

Step-by-step explanation: you know that opposite angles are the same. y=5

add 4 to both sides. and you get the value for x y z.

Can someone help me solve parts (a) and (c) please? Thank you!

Answers

a) 4x +6

Add up all the sides to calculate perimeter

Answer:

a) 6x + 6

b) 15 x 24

c) see explanation

Step-by-step explanation:

a) 2x + x + 3 + 2x + x + 3 = 6x + 6

b) 6x + 6 = 78

6x = 72

x = 12

2(12) = 24

(12) + 3 = 15

15 x 24

c) 2x(x + 3) = 2x² + 6x

2(12)² + 6(12) = 288 + 72 = 360

15 x 24 is also 360

Pls help, I don’t know how to fo

Answers

frustum of a cone is: = pi * l(R + r)

(l) = slant height of the frustum.

from 2929.645714 - 506.1257143

= 2423.52

= 2423.5cm

Answer:

from 2929.645714 - 506.1257143

= 2423.52

= 2423.5cm

Latanya buys 5 yard of blue fabric and 8 yards of green fabric. the blue fabric cost $2 dollars more than the green fabric.she pays a total of $ 62. what would be the combined cost of 1 yard of blue fabric and one yard of green fabric?

Answers

Answer: $10

Step-by-step explanation:

let x = the price of green fabric, then x+2 = blue fabric price

8x+5(x+2)=62

8x+5x+10=62

    13x+10=62

          13x=52

              x=4

price of green fabric=$4

price of blue fabric=$6

4+6=$10

A blimp is 1100 meters high in the air and measures the angles of depression to two stadiums to the west of the blimp. If those measurements are 75.2° and 17.9°, how far apart are the two stadiums?

Answers

Answer:

The two stadiums are approximately 3115.1 meters away from each other

Step-by-step explanation:

Since we can construct two right angle triangles between the blimp and the two stadiums as shown in the attached image, then the distance "x" between the two can be find as the difference between the right triangle legs that extend on the ground.

In order to find the size of such legs, one can use the tangent function of the given depression angles as shown below:

[tex]tan(75.2^o)=\frac{1100}{a} \\a=\frac{1100}{tan(75.2^o)}\\a\approx 290.6\,\,meters[/tex]

and for the other one:

[tex]tan(17.9^o)=\frac{1100}{b} \\b=\frac{1100}{tan(17.9^o)}\\b\approx 3405.7\,\,meters[/tex]

The the distance between the stadiums is the difference:

b - a = 3405.7  - 290.6 meters = 3115.1  meters

need help will give you a good rating us

Answers

Answer:

[tex]\boxed{2\pm \frac{\sqrt{2}}{2}}[/tex]

Step-by-step explanation:

[tex]2x^2-8x=-7[/tex]

[tex]\sf Add \ 7 \ on \ both \ sides}.[/tex]

[tex]2x^2-8x+7=-7+7[/tex]

[tex]2x^2-8x+7=0[/tex]

[tex]ax^2 +bx+c=0[/tex]

[tex]\sf Apply \ quadratic \ formula.[/tex]

[tex]a=2 \ \ \ b=-8 \ \ \ c = 7[/tex]

[tex]x=\frac{-b\pm\sqrt{b^2-4ac}}{2a}[/tex]

[tex]x=\frac{-(-8)\pm\sqrt{(-8)^2-4(2)(7)}}{2(2)}[/tex]

[tex]x=\frac{8\pm\sqrt{64-56}}{4}[/tex]

[tex]x=\frac{8\pm\sqrt{8}}{4}[/tex]

[tex]x=\frac{8\pm2\sqrt{2}}{4}[/tex]

[tex]x=2\pm \frac{\sqrt{2}}{2}[/tex]

ANSWER QUICKLY PLZZZZZZ
ANSWER QUESTION B​

Answers

Answer:

16

Step-by-step explanation:

We know that: 5 + ? = 10.5

we now need to solve for ? , by using the inverse method

10.5 x 2 = 21

21 - 5 = 16

Now to double check the answer, we substitute the ? with the answer

so, 5 + 16 = 21

      21/2 = 10.5  so it is clearly correct

I HOPE THIS HELPED :)

Please help me answer this question. -15 - g/3 = -5.
What is g?​

Answers

Answer:

g = -30

Step-by-step explanation:

-15 - g/3 = -5

Add 15 to each side

-15+15 - g/3 = -5+15

-g/3=10

Multiply by -3 to each side

-g/3 * -3  = 10*-3

g = -30

Answer:

g= -30

Step-by-step explanation:

after writing down the problem, multiply both sides of the equation by 3 to get  -45-g= =15. then subtract 15 from both sides to get g=-30, which is your answer. hope this helps!

Solve for x. 3x-91>-87 AND 17x-16>18

Answers

Answer & Step-by-step explanation:

For this problem, we have two inequalities to solve for x.

3x - 91 > -87

17x - 16 > 18

Now that we know what our inequalities are, we will solve them as if we are solving for the value of x.

3x - 91 > -87

Add 91 on both sides.

3x > 4

The solution for the first inequality is 3x > 4

Now let's do the second inequality.

17x - 16 > 18

Add 16 on both sides.

17x > 34

Divide by 17 on both sides.

x > 2

The soultion for the second inequality is x > 2

Answer:

The answer is x>2

Step-by-step explanation:

If you make $3.80 an hour plus tips, what is your paycheck for the week if you worked 40 hours and made $250.00 dollars in tips?

Answers

Answer:

$402

Step-by-step explanation:

Hello!

If you made 3.80 an hour and worked 40 we can multiply these to find the total amount you earned.

3.80 * 40 = 152

You also made 250 in tips so we add that to the total

152+250 = 402

The answer is $402

Hope this helps!

I would make $402.00 by the end of the week

What is the least common multiple of all positive integers smaller than 8?

Answers

Answer:

420

Step-by-step explanation:

We need to find the LCM of 1, 2, 3, 4, 5, 6 and 7. The LCM of 1, 2, and 4 is 4 and the LCM of 3 and 6 is 6 so the list becomes the LCM of 4, 5, 6 and 7. The LCM of 4 and 5 is 20, the LCM of 20 and 6 is 60 and the LCM of 60 and 7 is 420.

Answer:

420

Step-by-step explanation:

Prime factorization:

7 = 7

6 = 2 × 3

5 = 5

4 = 2 × 2

3 = 3

2 = 2

1 =  1

LCM: 7, 6, 5, 4, 3, 2, 1

2 × 2 × 3 × 5 × 7  = 420

-5x-10=10 solve for x

Answers

Answer:

x = -4

Step-by-step explanation:

-5x-10=10

Add 10 to each side

-5x-10+10=10+10

-5x = 20

Divide each side by -5

-5x/-5 = 20/-5

x = -4

Answer: x= -4

Step-by-step explanation:

[tex]-5x-10=10[/tex]

add 10 on both sides

[tex]-5x=20[/tex]

divide -5 on both sides

[tex]20/-5=-4[/tex]

[tex]x=-4[/tex]

Is 7 a solution of 5x - 3 = 12?

Answers

Answer:

no

Step-by-step explanation:

5x - 3 = 12

5x=12+3

x=15/5=3

x=3

substitute x with 7 to find if it is a solution :

5(7)-3=12

35-3 ≠12 ( so no 7 is not a solution)

If x3 + ax2 – bx + 10 is divisible by x2 – 3x + 2,
find the values of
1) a-b
2) 2a-b

Answers

Answer: A=2 and B=13
Explanation: The Factor Theorem states that if a is the root of any polynomial p(x) that is if p(a)=0, then (x−a) is the factor of the polynomial p(x).

Let p(x)=x
3
+ax
2
−bx+10 and g(x)=x
2
−3x+2
Factorise g(x)=x
2
−3x+2:
x
2
−3x+2=x
2
−2x−x+2=x(x−2)−1(x−2)=(x−2)(x−1)
Therefore, g(x)=(x−2)(x−1)
It is given that p(x) is divisible by g(x), therefore, by factor theorem p(2)=0 and p(1)=0. Let us first find p(2) and p(1) as follows:
p(1)=1
3
+(a×1
2
)−(b×1)+10=1+(a×1)−b+10=a−b+11
p(2)=2
3
+(a×2
2
)−(b×2)+10=8+(a×4)−2b+10=4a−2b+18
Now equate p(2)=0 and p(1)=0 as shown below:
a−b+11=0
⇒a−b=−11.......(1)
4a−2b+18=0
⇒2(2a−b+9)=0
⇒2a−b+9=0
⇒2a−b=−9.......(2)
Now subtract equation 1 from equation 2:

(2a−a)+(−b+b)=(−9+11)
⇒a=2
Substitute a=2 in equation 1:
2−b=−11
⇒−b=−11−2
⇒−b=−13
⇒b=13
Hence, a=2 and b=13.
Other Questions
What is the first rule Equality breaks, and where is he located when he breaks the first rule? Is 3+(-4) the same as -4+3 ? Explain Write a function called "equals" that accepts 2 arguments The two arguments will be of type list The function should return one string, either "Equals" or "Not Equals" For the lists to be equal, they need to: Have the same number of elements Have all the elements be of the same type Have the order fo the elements be the same DO NOT USE " HELP PLEASE ASAP I REALLY DONT KNOW HOW TO DO THIS What is the slope of a line perpendicular to the line containing the points $(-1,2)$ and $(1,-2)$? Express your answer as a common fraction. Enter your answer If a cell has 24 pairs of chromosomes in its diploid state, how manychromosomes will it have after Meiosis 2?A. 12B. 24C.48D. 6 (Pls Help) The study of such concepts as motion, force,y energy, matter, heat, sound, light, and the components of atoms is:- Physics- Biology- Earth Science- Astronomy Tarea: Escribe lo que opinas sobre las ideas/los sentimientos que t piensas que el autor est tratando de expresar en su poema. Ests de acuerdo o no? Por qu? A city of Punjab has a 15 percent chance of wet weather on any given day. What is the probability that it will take a week for it three wet weather on 3 separate days? Also find its Standard Deviation Read the following paragraph from Nevada Moose Herds Growing, an article written by Benjamin Spillman on page 1A in the October 18, 2017 edition of the Reno Gazette-Journal: Given the limited amount of viable moose habitat in Nevada, the driest state in the U.S., its likely the population is small. But the Nevada Department of Wildlife wants to alert the public to keep people from shooting them. Which of the options below most correctly and effectively incorporates this source material? (Either APA or MLA style citations are acceptable.) In his article Nevada Moose Herds Growing, Spillman notes that the Nevada Department of Wildlife doesnt want people shooting moose because their population is small (1A). Although the Nevada moose herds growing, that doesnt mean you should shoot them (Spillman 1A). Nevada is too dry to support a large moose herd, but their population is growing. (Nevada Department of Wildlife 1A). Its a sad state of affairs when NDOW announces a rare animal sighting not so we can enjoy them but to keep people from shooting them (Spillman 1A). explain how 6/12 and 4/8 are equal fractions The function f is defined by f(x) = (x + 3)(x + 1).The graph of f in the xy-plane is a parabola. Whichof the following intervals contains the x-coordinateof the vertex of the graph of f? Compare and contrast the BFG with the rest of the giants Which of the following statements is true for a function with equation Ax) = 5(3)*?The graph has y-intercept (0,5) and increases with a constant ratio of 3.The graph has y-intercept (0, 3) and decreases with a constant ratio of 3.The graph has y-intercept (0, 3) and increases with a constant ratio of 5.The graph has y-intercept (0,5) and decreases with a constant ratio of 3. Choose the correct sum of the polynomials (6x3 8x 5) + (3x3 + 6x + 2). 9x3 2x 3 3x3 + 2x + 3 3x3 2x 3 9x3 + 14x + 3 Milo Company manufactures beach umbrellas. The company is preparing detailed budgets for the third quarter and has assembled the following information to assist in the budget preparation: The Marketing Department has estimated sales as follows for the remainder of the year (in units): July 30,000 October 20,000 August 70,000 November 10,000 September 50,000 December 10,000 The selling price of the beach umbrellas is $12 per unit. All sales are on account. Based on past experience, sales are collected in the following pattern: 30% in the month of sale 65% in the month following sale 5% uncollectibleSales for June totaled $300.000. c. The company maintains finished goods inventories equal to 15% of the following month's sales. This requirement will be met at the end of June. d. Each beach umbrella requires 4 feet of Gilden, a material that is sometimes hard to acquire. Therefore, the company requires that the ending inventory of Gilden be equal to 5096 of the following month's production needs. The inventory of Gilden on hand at the beginning and end of the quarter will be: June 30 72,000 feetSeptember 30 ___ feete. Gilden costs $0.80 per foot. One-half of a month's purchases of Gilden is paid for in the month of purchase; the remainder is paid for in the following month. The accounts payable on July 1 for purchases of Gilden during June will be $76,000. Required: 1. Prepare a sales budget, by month and in total, for the third quarter. (Show your budget in both units and dollars.) Also prepare a schedule of expected cash collections, by month and in total, for the third quarter. 2. Prepare a production budget for each of the months July-October. 3. Prepare a direct materials budget for Gilden, by month and in total, for the third quarter. Also prepare a schedule of expected cash disbursements for Gilden, by month and in total, for the third quarter. Factorise : x^2-9x-70 Step by Step How would you simplify and rationalize this expression? [tex]\frac{5\sqrt[4]{2}}{4\sqrt[4]{162} }[/tex] PLS HELP ASAP Solve the inequality and enter your solution as an inequality in the box below 8>4-x>6 What is the solution to -5 + z = -12 A. z = -17 B. z = -7 C. z = 7 D. z = 17 he most common source of osteomyelitis is an infection that migrates via the bloodstream. direct invasion from a fracture. surgical contamination. a joint prosthesis.